Đến nội dung

nhungvienkimcuong

nhungvienkimcuong

Đăng ký: 23-12-2014
Offline Đăng nhập: Riêng tư
****-

#562850 $f(f(x))=x^2-2$

Gửi bởi nhungvienkimcuong trong 01-06-2015 - 15:48

Chứng minh không tồn tại hàm số $f(x)$ xác định với mọi số thực $x$ và thỏa mãn $f(f(x))=x^2-2$ với mọi x

ta chứng minh bài toán tổng quát sau

$\blacksquare$ $\boxed{\text{Tổng quát hóa}}$

Cho $S$ là một tập hợp và $g:S\rightarrow S$ có chính xác hai điểm cố định $\left \{ a,b \right \}$ và $g\circ g$ có chính xác $4$ điểm cố định $\left \{ a,b,c,d \right \}$

CMR không tồn tại hàm số $f:S\rightarrow S$ sao cho $g=f\circ f$

$\triangleright$ Chứng minh 

đặt $y=g(c)\Rightarrow g(y)=g\left ( g(c) \right )=c\Rightarrow y=g(c)=g(g(y))$

$\Rightarrow$ $y$ là một điểm cố định của $g\circ g$

$-$ với $g(c)=a\Rightarrow a=g(a)=g(g(c))=c$ $($ vô lí $)$

$-$ với $g(c)=b$ tương tự như trên ta có mâu thuẫn

$-$ với $g(c)=c$ thì $c$ là điểm cố định của $g$ $($ vô lí $)$

do đó ta có $g(c)=d$ và tương tự như vậy ta có $g(d)=c$

giả sử tồn tại hàm $f$ thỏa đề rằng $g=f\circ f$ do đó ta có $f\circ g=f\circ f\circ f=g\circ f$

$\Rightarrow f(a)=f(g(a))=g(f(a))$

$\Rightarrow$ $f(a)$ là một điểm bất động của $g$ do đó $f\left \{ a,b \right \}=\left \{ a,b \right \}$

mặt khác $f(c)=f\left ( g\left ( g(c) \right ) \right )=g(f(g(c)))=g(g(f(c)))$

$\Rightarrow$ $f(c)$ là một điểm bất động của $g\circ g$ nên $f\left \{ a,b,c,d \right \}=\left \{ a,b,c,d \right \}$

giờ ta xét $f(c)$

$-$ nếu $f(c)=a\Rightarrow f(a)=f(f(c))=g(c)=d$ $($ mâu thuẫn $)$

$-$ nếu $f(c)=b$ thì tương tự như trên ta cũng có mâu thuẫn

$-$ nếu $f(c)=c\Rightarrow c=f(c)=f(f(c))=g(c)$ $($ mâu thuẫn với giả thiết $)$

$-$ nếu $f(c)=d\Rightarrow f(d)=f(f(c))=g(c)=d$ thì tương tự như trên ta cũng có điều vô lí

do đó không tồn tại hàm $f$ thỏa đề $\blacksquare$

 

$\blacksquare$ $\boxed{\text{Quay lại bài toán}}$

áp dụng bài toán trên với $g(x)=x^2-2$ có $2$ điểm bất động là $\left \{ -1,2 \right \}$

                                   và $g(g(x))=(x^2-2)^2-2$ có $4$ điểm bất động là $\left \{ -1,2,\frac{-1+\sqrt{5}}{2},\frac{-1-\sqrt{5}}{2} \right \}$

Do đó không tồn tại hàm $f$ thỏa đề $\blacksquare$




#562311 $(XYZ)$ tiếp xúc với $(ABC)$

Gửi bởi nhungvienkimcuong trong 29-05-2015 - 19:54

sao hai đường tròn này trực giao nhỉ

vì $(U)$ là đường tròn $\text{Apollonius}$ của $\Delta YIT$




#562265 ĐỀ THI OLYMPIC CHUYÊN KHOA HỌC TỰ NHIÊN NĂM 2015

Gửi bởi nhungvienkimcuong trong 29-05-2015 - 14:20

cho mình hỏi $v_{p}(...)$ là gì vậy

đó là kí hiệu cho số mũ nguyên tố đúng

còn định lý $\text{Poncelet}$ bạn tham khảo thêm ở đây




#562242 $(XYZ)$ tiếp xúc với $(ABC)$

Gửi bởi nhungvienkimcuong trong 29-05-2015 - 11:15

Cho tam giác $ABC$ ngoại tiếp đường tròn $(I)$. Đường tròn $(T)$ ngoại tiếp tam giác $AIB$, Hai đường tròn này giao nhau tại $X,Y$. Gọi $Z$ là giao điểm hai tiếp tuyến chung của $(I),(T)$. Chứng minh rằng: đường tròn ngoại tiếp tam giác $XYZ$ tiếp xúc đường tròn ngoại tiếp tam giác $ABC$

Capture.PNG

kí hiệu $R_A$ là bán kính đường tròn $(A)$

lời giải

Đặt $T=CI\cap (ABC)$ thì dễ thấy $T$ là tâm của $(AIB)$

Gọi $P$ là tâm vị tự trong của $(I)$ và $(T)$,$U$ là tâm $(XYZ)$

Ta có

$\frac{ZI}{ZT}=\frac{PI}{PT}=\frac{R_I}{R_T}=\frac{YI}{YT}=\frac{XI}{XT}$

Do đó $X,Y,Z,P$ cùng nằm trền đường tròn $\text{Apollonius}$ của $\Delta YIT$

$\Rightarrow$ $(U)$ và $(YIT)$ trực giao nên $UY$ là tiếp tuyến của $(YIT)$

$\Rightarrow \widehat{UYI}=\widehat{YTI}\Rightarrow \widehat{UYT}+\widehat{IYT}=\widehat{ITU}+2\widehat{IYT}=180^0$

$\Rightarrow$ $YT$ là phân giác ngoài $\Delta UIY \Rightarrow \frac{TI}{TU}=\frac{R_I}{R_U}$

$\Rightarrow$ $T$ là tâm vị tự ngoài của $(I)$ và $(U)$

Kẻ tiếp tuyến $TM,TN$ với $(U)$ và chúng lần lượt cắt $(ABC)$ ở $L,K$

Ta có $(ZPIT)=-1$ mà $U$ là trung điểm $ZP$ 

$\Rightarrow UM^2=UZ^2=UI.UT\Rightarrow$ $I$ là hình chiếu của $M$ trên $UT$

$\Rightarrow$ ta chứng minh được $I$ là trung điểm $MN$

Theo định lý $\text{Poncelet}$ thì $(I)$ cũng là đường tròn nội tiếp $\Delta TLK$

Do đó theo định lý $\text{Sawayama}$ thì $(U)$ là đường tròn $\text{mixtilinear}$ nội tiếp ứng với góc $T$ của $\Delta TLK$

$\Rightarrow$ $(U)$ tiếp xúc với $(TLK)$ hay ta có $Q.E.D$

Spoiler




#562234 Tổng hợp các định lý trong hình học và ứng dụng của chúng qua các bài toán

Gửi bởi nhungvienkimcuong trong 29-05-2015 - 10:19

Mọi người chứng minh giúp em định lý Poncelet

Cho tam giác ABC nội tiếp (O) ngoại tiếp (I). Lấy một điểm D thuộc (O) và kẻ 2 tiếp tuyến DE, DF của (I). chứng minh rằng EF cũng là tiếp tuyến của (I)

anh xem tài liệu sau với hệ quả $2.2$

File gửi kèm  poncelet.pdf   173.92K   1000 Số lần tải




#561853 CMR: Giữa 2 số $\sum_{i=1}^{n}p_{i}...

Gửi bởi nhungvienkimcuong trong 27-05-2015 - 10:34

 

Cho $p_{1}<p_{2}<...<p_{n}<p_{n+1}$ là các số nguyên tố, CMR:
Giữa 2 số $\sum_{i=1}^{n}p_{i}$ và $\sum_{i=1}^{n+1}p_{i}$ có 1 số chính phương

 

xem ở đây

với cách chứng minh ở trên ta cũng chứng minh được bài toán tổng quát hơn như sau

$\boxed{\text{Problem}}$ $($ Đề chọn đội tuyển $30/4$ chuyên Lê Hồng Phong $2014-2015$ $)$

Cho $k_1<k_2<...<k_n$ là các số nguyên dương tron đó không có hai số liên tiếp.Đặt $S_n=k_1+k_2+...+k_n$.Chứng minh rằng giữa hai số $S_n$ và $S_{n+1}$ có ít nhất một số chính phương




#561793 CMR có vô số số nguyên dương $n$ để $[n\sqrt{2}...

Gửi bởi nhungvienkimcuong trong 26-05-2015 - 22:50

CMR có vô số số nguyên dương $n$ để $[n\sqrt{2}]$ là số chính phương

xem ở đây




#561729 $(a+b+c)(\frac{1}{a}+\frac{1}...

Gửi bởi nhungvienkimcuong trong 26-05-2015 - 19:28

Tìm k lớn nhất sao cho:

$(a+b+c)(\frac{1}{a}+\frac{1}{b}+\frac{1}{c})+k.\frac{ab+ac+bc}{a^{2}+b^{2}+c^{2}}\geqslant 9+k$

xem ở đây

Spoiler




#561709 Topic tổng hợp một số bất đẳng thức trong kì thi MO các nước

Gửi bởi nhungvienkimcuong trong 26-05-2015 - 18:13

Bài 57:(Áo 1971)

$a,b,c$ là các số dương thỏa $a\leq b\leq c$ và $x,y,z$ là các số thực dương tùy ý.Chứng minh rằng

$\frac{(a+c)^2}{4ac}(x+y+z)^2\geq (ax+by+cz)\left ( \frac{x}{a}+\frac{y}{b}+\frac{z}{c} \right )$

 

Bài 58:(Iran 2010)

Cho ba số dương $x,y,z$ thỏa $xy+yz+zx=1$.Chứng minh rằng

$\frac{x^2}{y}+\frac{y^2}{z}+\frac{z^2}{x}+3\geq (x+y+z)^2+\sqrt{3}$




#561682 Chứng minh rằng: $2(x+y+z)-xyz\leqslant 10$

Gửi bởi nhungvienkimcuong trong 26-05-2015 - 15:13

P.s. Dùng kỹ thuật phân chia trường hợp

bài toán kinh điển cho kĩ thuật này là

$\boxed{\text{Problem}}$

Với $a,b,c$ là các số thực dương.Chứng minh rằng

$\frac{1}{(a+2b)^2}+\frac{1}{(b+2c)^2}+\frac{1}{(c+2a)^2}\geq \frac{1}{ab+bc+ca}$




#561675 CMR $\begin{pmatrix} n\\2014p \end{pm...

Gửi bởi nhungvienkimcuong trong 26-05-2015 - 14:23

$\boxed{\text{Problem 1}}$

Cho $p$ là số nguyên tố lớn hơn $2015$ và $n$ là số nguyên dương lớn hơn $2014p$.Đặt $k=\left [ \frac{n}{p} \right ]$

Chứng minh rằng

$\begin{pmatrix} n\\2014p \end{pmatrix}\equiv \begin{pmatrix} k\\2014 \end{pmatrix}(mod\ p)$

 

$\boxed{\text{Problem 2}}$

Cho $p$ là số nguyên tố và $a,b,x,y$ là các số tự nhiên với $a>b>0,0\leq y\leq x<p$

Chứng minh rằng

$\begin{pmatrix} pa+x\\pb+y \end{pmatrix}\equiv \begin{pmatrix} a\\b \end{pmatrix}\begin{pmatrix} x\\y \end{pmatrix}(mod\ p)$




#561647 Topic tổng hợp một số bất đẳng thức trong kì thi MO các nước

Gửi bởi nhungvienkimcuong trong 26-05-2015 - 09:10

Bài 55:(Ailen 2009)

Cho $a,b,c$ là các số thực thỏa $\left\{\begin{matrix} a+b+c=0\\a^2+b^2+c^2=1 \end{matrix}\right.$

Chứng minh rằng

$a^2b^2c^2\leq \frac{1}{54}$

 

Bài 56:(Ấn Độ 2009)

Cho $a,b,c>0$ và thỏa mãn $a^3+b^3=c^3$.

Chứng minh rằng

$a^2+b^2-c^2>6(a-c)(b-c)$




#561370 Topic tổng hợp một số bất đẳng thức trong kì thi MO các nước

Gửi bởi nhungvienkimcuong trong 24-05-2015 - 18:59

Bài 53:
$a^2+b^2+c^2=3$ nên $abc \leq 1$
$\frac{1}{a^3b}+\frac{1}{b^3c}+\frac{1}{c^3a} \geq 3\sqrt[3]{\frac{1}{a^4b^4c^4}} \geq 3 $

làm phiền bạn nhưng mình đăng đề nhầm

xin lỗi




#561367 CMR $\sum_{i=1}^{\frac{p-1}{2...

Gửi bởi nhungvienkimcuong trong 24-05-2015 - 18:48

Cho $p$ là số nguyên tố lẻ.CMR

$1^{p-2}+2^{p-2}+...+\left ( \frac{p-1}{2} \right )^{p-2}\equiv \frac{2-2^p}{p}(mod\ p)$

 




#561366 Topic tổng hợp một số bất đẳng thức trong kì thi MO các nước

Gửi bởi nhungvienkimcuong trong 24-05-2015 - 18:44

Bài 52:(IMO Shortlist 2009)

Cho $a,b,c>0$ mà $\frac{1}{a}+\frac{1}{b}+\frac{1}{c}\leq 3$.CMR

$\sqrt{\frac{a^2+b^2}{a+b}}+\sqrt{\frac{b^2+c^2}{b+c}}+\sqrt{\frac{c^2+a^2}{c+a}}+3\leq \sqrt{2}\left ( \sqrt{a+b}+\sqrt{b+c}+\sqrt{c+a} \right )$

 

Spoiler

Spoiler